Difference between revisions of "2020 AMC 12B Problems/Problem 4"

(Created page with "==Problem== The acute angles of a right triangle are <math>a^\circ</math> and <math>b^\circ</math>, where <math>a>b</math> and both <math>a</math> and <math>b</math> are prime...")
 
Line 4: Line 4:
  
 
==Solution==
 
==Solution==
 
+
<math>a+b+90=180</math>, so <math>a+b=90</math>. The largest primes less than <math>90</math> are <math>89, 83, 79, ...</math> If <math>a=89</math>, then <math>b=1</math>, which is not prime. However, if <math>a=83</math>, then <math>b=7</math>, which is prime. Hence the answer is <math>\boxed{\mathrm{(D)}}</math>
 
==See Also==
 
==See Also==
  
 
{{AMC12 box|year=2020|ab=B|num-b=3|num-a=5}}
 
{{AMC12 box|year=2020|ab=B|num-b=3|num-a=5}}
 
{{MAA Notice}}
 
{{MAA Notice}}

Revision as of 21:08, 7 February 2020

Problem

The acute angles of a right triangle are $a^\circ$ and $b^\circ$, where $a>b$ and both $a$ and $b$ are prime numbers. What is the least possible value of $b$?


Solution

$a+b+90=180$, so $a+b=90$. The largest primes less than $90$ are $89, 83, 79, ...$ If $a=89$, then $b=1$, which is not prime. However, if $a=83$, then $b=7$, which is prime. Hence the answer is $\boxed{\mathrm{(D)}}$

See Also

2020 AMC 12B (ProblemsAnswer KeyResources)
Preceded by
Problem 3
Followed by
Problem 5
1 2 3 4 5 6 7 8 9 10 11 12 13 14 15 16 17 18 19 20 21 22 23 24 25
All AMC 12 Problems and Solutions

The problems on this page are copyrighted by the Mathematical Association of America's American Mathematics Competitions. AMC logo.png